Difference between revisions of "1969 AHSME Problems/Problem 35"
(Created page with "== Problem == Let <math>L(m)</math> be the <math>x</math> coordinate of the left end point of the intersection of the graphs of <math>y=x^2-6</math> and <math>y=m</math>, where ...") |
Rockmanex3 (talk | contribs) (Solution to Problem 35 (credit to soy_un_chemisto and tim9099xxzz) -- calculus problem on non-calculus test) |
||
Line 9: | Line 9: | ||
\text{(E) undetermined} </math> | \text{(E) undetermined} </math> | ||
− | == | + | == Solutions == |
− | |||
− | == See | + | ===Solution 1=== |
− | {{AHSME 35p box|year=1969|num-b=34| | + | |
+ | Since <math>L(a)</math> is the <math>x</math> coordinate of the left end point of the intersection of the graphs of <math>y=x^2-6</math> and <math>y=a</math>, we can substitute <math>a</math> for <math>y</math> and find the lowest solution <math>x</math>. | ||
+ | <cmath>x^2 - 6 = a</cmath> | ||
+ | <cmath>x = \pm \sqrt{a+6}</cmath> | ||
+ | That means <math>L(m) = -\sqrt{m+6}</math> and <math>L(-m) = -\sqrt{-m+6}</math>. That means | ||
+ | <cmath>r = \frac{-\sqrt{-m+6}+\sqrt{m+6}}{m}</cmath> | ||
+ | Since plugging in <math>0</math> for <math>m</math> results in <math>0/0</math>, there is a removable discontinuity. Multiply the fraction by <math>\frac{\sqrt{-m+6}+\sqrt{m+6}}{\sqrt{-m+6}+\sqrt{m+6}}</math> to get | ||
+ | <cmath>r = \frac{m+6 - (-m+6)}{m(\sqrt{-m+6}+\sqrt{m+6})}</cmath> | ||
+ | <cmath>r = \frac{2m}{m(\sqrt{-m+6}+\sqrt{m+6})}</cmath> | ||
+ | <cmath>r = \frac{2}{(\sqrt{-m+6}+\sqrt{m+6})}</cmath> | ||
+ | Now there wouldn't be a problem plugging in <math>0</math> for <math>m</math>. Doing so results in <math>r = \tfrac{2}{2\sqrt{6}} = \tfrac{1}{\sqrt{6}}</math>, so the answer is <math>\boxed{\textbf{(B)}}</math>. | ||
+ | |||
+ | ===Solution 2=== | ||
+ | |||
+ | From Solution 1, <math>L(m) = -\sqrt{m+6}</math> and <math>L(-m) = -\sqrt{-m+6}</math>, so | ||
+ | <cmath>r = \frac{-\sqrt{-m+6}+\sqrt{m+6}}{m}</cmath> | ||
+ | Since <math>m</math> is arbitrarily close to <math>0</math>, we wish to find | ||
+ | <cmath>\lim_{m \rightarrow 0} \frac{-\sqrt{-m + 6} + \sqrt{m + 6}}{m}</cmath> | ||
+ | Using [[L'Hopital's Rule]], the limit is equivalent to | ||
+ | <cmath>\lim_{m \rightarrow 0} \frac{1}{2 \sqrt{-m + 6}} + \frac{1}{2 \sqrt{m + 6}}</cmath> | ||
+ | Calculating the limit shows that <math>r</math> is <math>\boxed{\text{(B) arbitrarily close to }\frac{1}{\sqrt{6}}}</math>. | ||
+ | |||
+ | == See Also == | ||
+ | {{AHSME 35p box|year=1969|num-b=34|after=Last Problem}} | ||
[[Category: Intermediate Algebra Problems]] | [[Category: Intermediate Algebra Problems]] | ||
{{MAA Notice}} | {{MAA Notice}} |
Latest revision as of 12:19, 27 July 2018
Problem
Let be the coordinate of the left end point of the intersection of the graphs of and , where . Let . Then, as is made arbitrarily close to zero, the value of is:
Solutions
Solution 1
Since is the coordinate of the left end point of the intersection of the graphs of and , we can substitute for and find the lowest solution . That means and . That means Since plugging in for results in , there is a removable discontinuity. Multiply the fraction by to get Now there wouldn't be a problem plugging in for . Doing so results in , so the answer is .
Solution 2
From Solution 1, and , so Since is arbitrarily close to , we wish to find Using L'Hopital's Rule, the limit is equivalent to Calculating the limit shows that is .
See Also
1969 AHSC (Problems • Answer Key • Resources) | ||
Preceded by Problem 34 |
Followed by Last Problem | |
1 • 2 • 3 • 4 • 5 • 6 • 7 • 8 • 9 • 10 • 11 • 12 • 13 • 14 • 15 • 16 • 17 • 18 • 19 • 20 • 21 • 22 • 23 • 24 • 25 • 26 • 27 • 28 • 29 • 30 • 31 • 32 • 33 • 34 • 35 | ||
All AHSME Problems and Solutions |
The problems on this page are copyrighted by the Mathematical Association of America's American Mathematics Competitions.